Задачи

Фильтрация

Показать фильтрацию

По классам:

По предметам:

По подготовке:

По классам:

По авторам:

Докажите, что последовательность \(\left \{ x_{n} \right \}\) сходится: \(x_{n}=\frac{n^{3}}{10^{n}} \)

Решение №13752: Покажем, что последовательность убывает: \(\frac{x_{n+1}}{x_{n}}=\frac{\left ( n+1 \right )^{3}*10^{n}}{10^{n+1}*n^{3}}=\left ( \frac{n+1}{n} \right )^{3}*\frac{1}{10}< 1.\) Так как \(\forall n\in N \frac{n+1}{n}\leqslant 2\), а значит, \(\left ( \frac{n+1}{n} \right )^{3}< 10\). Таким образом \(\forall n\in N x_{n+1}< x_{n}\), т.е. последовательность \(\left \{ x_{n} \right \}\)убывает и ограничена снизу нулем \(\left ( \forall n\in N x_{n}> 0 \right ).\) По теореме Вейерштрасса она имеет предел. Можно показать, как найти этот предел (этого не требуется в задаче). Пусть\(\lim_{n \to \propto} x_{n}=\lim_{n \to \propto} x_{n+1}=a\), имеем \(x_{n+1}=\frac{1}{10}\left ( \frac{n+1}{n} \right )^{3}x_{n}\). Так как \(\lim_{n \to \propto} \left ( \frac{n+1}{n} \right )^{3}=1, a=\frac{1}{10}a\). Значит a=0.

Ответ: NaN

Докажите, что последовательность \(\left \{ x_{n} \right \}\) сходится: \(x_{n}=\frac{2^{n}}{n!} \)

Решение №13753: \( \forall n> 2 \frac{x_{n+1}}{x_{n}}=\frac{2}{n+1}< 1\). Доказательство того, что предел существует, здесь гораздо очевиднее, но зато нет способа его найти. Предельный переход, как в предыдущем пункте, здесь ничего не даёт для поиска предела последовательности \(\left \{ x_{n} \right \}\), равного a

Ответ: NaN

Докажите, что последовательность \(\left \{ x_{n} \right \}\) сходится: \(x_{1}=8, x_{2}=\frac{8}{1}*\frac{11}{7}, ..., x_{n}=\frac{8}{1}*\frac{11}{7}* ...*\frac{3n+5}{6n-5} \)

Решение №13754: \( \forall n\in N \frac{x_{n+1}}{x_{n}}=\frac{3n+8}{6n+1}\). Так как \(\lim_{n \to \propto}\frac{x_{n+1}}{x_{n}}=\frac{1}{2}< 1 \forall n\in N x_{n}> 0\). А можно так:\( \forall n> 8 \frac{3n+8}{6n+1}< \frac{2}{3} \) Значит, при n> 8 последовательность \(\left \{ x_{n} \right \}\) убывает и ограничена снизу нулем. Следовательно, существует \( \lim_{n \to \propto} x_{n} \)

Ответ: NaN

Докажите, что \(\left \{ x_{n} \right \}\) сходится, и найдите \(\lim_{n \to \propto} x_{n} : x_{1}=\sqrt{a}, x_{n+1}=\sqrt{a+x_{n}}\), где \(a> 0\)

Решение №13757: Выясним сначала, чему может быть равен предел последовательности \(\left \{ x_{n} \right \}\). Пусть \(\lim_{n \to \propto} x_{n}=B, тогда B=\sqrt{B+a}, B^{2}-B-a=0 B=\frac{1+\sqrt{1+4a}}{2} (B> 0)\), так как \(\forall n\in N x_{n}> 0)\). Докажем по индукции, что \(\forall n\in N x_{n}< x_{n+1}< B\). База индукции очевидна. Индукционный переход. Докажем, что \(x_{k}< x_{k+1}< B\), если \(x_{k-1}< x_{k}< B\). Действительно, \(x_{k-1}< x_{k}< B\Leftrightarrow x_{k-1}< \sqrt{a+x_{k-1}}< B\), но в силу монотонности корня и того, что \(x_{k-1}< x_{k}\), выполняется \(\sqrt{a+x_{k-1}}< \sqrt{a+x_{k}=x_{k+1}}< B (так как x_{k}< B \sqrt{a+x_{k}}< \sqrt{a+B}=B)\). Мы доказали, что последовательность \(\left \{ x_{n} \right \}\\) возрастает и ограничена снизу и имеет предел, равный \(\frac{1+\sqrt{1+4a}}{2} \)

Ответ: NaN

Докажите, что \(\left \{ x_{n} \right \}\) сходится, и найдите \(\lim_{n \to \propto} x_{n} : x_{n+1}=\frac{4}{3}x_{n}-x_{n}^{2}, 1) x_{1}=\frac{1}{6}; 2)x_{1}=\frac{1}{2}; x_{3}=\frac{7}{6}\)

Решение №13759: Докажем для случая \(x_{1}=\frac{1}{6}\). Рассмотрим разность \(x_{n+1}-x_{n}=\frac{1}{3x_{n}}-x_{n}^{2}=x_{n}\left ( \frac{1}{3}-x_{n} \right )\). По индукции легко показать, что последовательность \(\left \{ x_{n} \right \}\) возрастает и\(\forall n\in N x_{n}\leqslant \frac{1}{3}\). База индукции \(x_{1}=\frac{1}{6}. Тогда x_{2}-x_{1}=\frac{1}{6}\left ( \frac{1}{3}-\frac{1}{6} \right )=\frac{1}{36}> 0\) и \(x_{2}=x_{1}+\frac{1}{36}=\frac{13}{36}, 0< x_{1}< x_{2}< \frac{1}{3}\). Индукционный переход. Докажем, что \(0< x_{k}< x_{k+1}< \frac{1}{3},если 0< x_{k-1}< x_{k}< \frac{1}{3}\). Так как \(x_{k+1}-x_{k}=x_{k}\left ( \frac{1}{3} -x_{k}\right )> 0\) (по индукционному предположению),\(x_{k+1}> x_{k}\). Если рассмотреть функцию \(g\left ( t \right )=t\left ( \frac{4}{3}-t \right )-\frac{1}{3}; g\left ( t \right )=-\frac{1}{3}\left ( 3t-1 \right )\left ( t-1 \right ), g\left ( t \right )> 0\Leftrightarrow \frac{1}{3}< t< 1\), поэтому \(g\left ( x \right )< 0, x_{k+1}< \frac{1}{3} \)

Ответ: NaN

Выясните, сходится ли последовательность \(\left \{ x_{n} \right \}\) и найдите предел сходящейся последовательности: \(x_{1}\leqslant 1, x_{n+1}=-\sqrt{1-x_{n}}, где n\in N \)

Решение №13760: Докажем, что последовательность \(\left \{ x_{n} \right \} \)убывает и ограничена снизу нулем. \(0\leqslant x_{2}=x_{1}\left ( 1-x_{1} \right )\leqslant x_{1}\leqslant 1\), так как \(0\leqslant x_{1}\leqslant 1\). Легко показать по индукции, что \(0\leqslant x_{k+1}=x_{k}\left ( 1-x_{k} \right )\leqslant x_{k}\) (индукция нужна лишь для доказатнльства неравенства \(x_{k+n}\geqslant 0)\). Тогда пусть \(\lim_{n \to \propto} x_{n}=\lim_{n \to \propto} x_{n+1}=a\). Итак \(a=a-a^{2}\Leftrightarrow a=0. \)

Ответ: NaN

Найдите\( \lim_{n \to \propto} x_{n}, если x_{n}=\left ( \frac{n}{n+1} \right )^{n}\)

Решение №13764: \( \lim_{n \to \propto} \left ( \frac{n}{n+1} \right )^{n}=\lim_{n \to \propto}\frac{1}{\left ( 1+\frac{1}{n} \right )^{n}}=e^{-1} \)

Ответ: e^{-1}

Дана последовательность\( \left \{ a_{n} \right \}\). Рассмотри последовательности \(x_{n}=a_{2n}, y_{n}=a_{2n-1}, z_{n}=a_{2n+4}, u_{n}=a_{3n}\) Верно ли утверждение, что последовательность\( \left \{ x_{n} \right \}\) сходится, то и последовательность\( \left \{ z_{n} \right \}\) сходится?

Пока решения данной задачи,увы,нет...

Ответ: Нет

Дана последовательность \(\left \{ a_{n} \right \}\). Рассмотри последовательности \(x_{n}=a_{2n}, y_{n}=a_{2n-1}, z_{n}=a_{2n+4}, u_{n}=a_{3n}\). Очевидно, что если последовательность \(\left \{ a_{n} \right \}\) сходится, то и сходятся последовательности \(\left \{ x_{n} \right \} и \left \{ y_{n} \right \}\). Верно ли утверждение: если последовательности \(\left \{ x_{n} \right \} и \left \{ y_{n} \right \}\) сходятся, то и последовательность \(\left \{ a_{n} \right \} \)сходится?

Пока решения данной задачи,увы,нет...

Ответ: Нет, например a_{n}=\left ( -1 \right )^{n}